Difference between revisions of "G285 MC10B Problems/Problem 1"

m (Solution)
m
Line 5: Line 5:
  
 
==Solution==
 
==Solution==
We have <cmath>\frac{6+24+120+720}{20} = \frac{87}{2} = \lfloor 43.5 \rfloor \implies \boxed{\textbf{(B)}\ 43}</cmath>
+
We have <cmath>\frac{6+24+120+720}{20} = \frac{87}{2} = \lceil 43.5 \rceil \implies \boxed{\textbf{(C)}\ 44}</cmath>
  
{{MC10B box|year=2021|ab=B|num-b=1|after=2}}
+
{{AMC10 box|year=2021|ab=B|num-b=1|after=2}}

Revision as of 20:33, 20 June 2021

Problem

Find $\left \lceil {\frac{3!+4!+5!+6!}{2+3+4+5+6}} \right \rceil$

$\textbf{(A)}\ 42\qquad\textbf{(B)}\ 43\qquad\textbf{(C)}\ 44\qquad\textbf{(D)}\ 45\qquad\textbf{(E)}\ 46$

Solution

We have \[\frac{6+24+120+720}{20} = \frac{87}{2} = \lceil 43.5 \rceil \implies \boxed{\textbf{(C)}\ 44}\]

2021 AMC 10B (ProblemsAnswer KeyResources)
Preceded by
Problem 1
Followed by
2
1 2 3 4 5 6 7 8 9 10 11 12 13 14 15 16 17 18 19 20 21 22 23 24 25
All AMC 10 Problems and Solutions